Themenbereiche Themenbereiche Profile Hilfe/Anleitungen Help    
Recent Posts Last 1|3|7 Days Suche Suche Tree Tree View  

Feiertagsproblem

ZahlReich - Mathematik Hausaufgabenhilfe » ---- Archiv: Universitäts-Niveau » Zahlentheorie » Feiertagsproblem « Zurück Vor »

Autor Beitrag
Seitenanfangvoriger Beitragnächster BeitragSeitenende Link zu diesem Beitrag

Tilo Kruse (bbk)
Suche alle Beiträge dieser Person in dieser Hauptrubrik
Mitglied
Benutzername: bbk

Nummer des Beitrags: 17
Registriert: 12-2002
Veröffentlicht am Samstag, den 28. Dezember, 2002 - 17:53:   Beitrag drucken

Habe auch die Feiertage Probleme mit einer Aufgabe:

4 + 4 + 4 + 4 + 4 = 20 ergibt auf Spanisch:
CUATRO + CUATRO + CUATRO + CUATRO + CUATRO = VEINTE
Wie kann man die zehn Ziffern den zehn Buchstaben A, C, E, I, N, O, R, T, U und V so (bijektiv) zordnen, dass die Addition stimmt?
Geben sie nachvollziehbart an, wie sie zu ihrem Ergebnis kommen.

Mir ist folgendes klar:
C muss 1 sein
E muss 0 oder 5 sein

Das ist aber leider schon alles, kann mir jemand weiterhelfen?

P.S. Guten Rutsch an alle
Seitenanfangvoriger Beitragnächster BeitragSeitenende Link zu diesem Beitrag

Tilo Kruse (bbk)
Suche alle Beiträge dieser Person in dieser Hauptrubrik
Mitglied
Benutzername: bbk

Nummer des Beitrags: 18
Registriert: 12-2002
Veröffentlicht am Montag, den 30. Dezember, 2002 - 19:33:   Beitrag drucken

Bitte Hilfe...
Seitenanfangvoriger Beitragnächster BeitragSeitenende Link zu diesem Beitrag

Chris (rothaut)
Suche alle Beiträge dieser Person in dieser Hauptrubrik
Mitglied
Benutzername: rothaut

Nummer des Beitrags: 31
Registriert: 01-2001
Veröffentlicht am Dienstag, den 31. Dezember, 2002 - 11:00:   Beitrag drucken

Tach Tilo,

ich hab mal ein bischen rumgekniffelt und mich an den Logiktrainer aus PM erinnert...

Also fangen wir mal an:

Um das nachzuvollziehen schreib Dir mal die verbleibenden Möglichkeiten der Buchstaben auf. z.B a={0,1,2,3,4,5,6,7,8,9} und dann streichst Du das weg, was nicht mehr sein kann.

Ich nehme an c=1 und für e=0 oder 1 (klar)

Da an der 2. Stelle im Ergebnis auch ein E steht, kann A keinen Übertrag haben -> a=0 -> e=5
-> o und u ungerade .
( Jetzt ist z.B. v={6,8,9} und u={3,7,9} )
U kann nicht 9 sein, denn dann wäre v auch 9
so kannst Du jetzt weitermachen...
z.B. Da a=0 -> I ist Übertrag von T -> I={2,3,4}
usw. usw.

nachher hatte ich v={6,8} und wenn ich 8 ausprobiere ergeben sich die anderen zahlen ohne Widerspruch.

Ergebnis zur Kontrolle:
c=1, u=7, a=0, t=4, r=6, o=9, v=8, e=5, i=2, n=3

5*170469=852345 (stimmt)
Seitenanfangvoriger Beitragnächster BeitragSeitenende Link zu diesem Beitrag

Tilo Kruse (bbk)
Suche alle Beiträge dieser Person in dieser Hauptrubrik
Mitglied
Benutzername: bbk

Nummer des Beitrags: 19
Registriert: 12-2002
Veröffentlicht am Dienstag, den 31. Dezember, 2002 - 18:51:   Beitrag drucken

Danke, werde es mir nach Sylvester mal genauer anschauen...guten Rutsch und nochmal danke!
Seitenanfangvoriger Beitragnächster BeitragSeitenende Link zu diesem Beitrag

Tilo Kruse (bbk)
Suche alle Beiträge dieser Person in dieser Hauptrubrik
Mitglied
Benutzername: bbk

Nummer des Beitrags: 20
Registriert: 12-2002
Veröffentlicht am Mittwoch, den 01. Januar, 2003 - 17:44:   Beitrag drucken

Hm...es sind leider dochn Fragen aufgetaucht
So weit habe ich es nun reduzieren können:

A {0}
C {1}
E {5}
I {2,3,4}
N {2,3,4,6,7,8,9}
O {3,7,9}
R {2,3,4,6,7,8,9}
T {4,6,7,8}
U {3,7}
V {6,8}

Was muss ich jetzt bedenken?
Seitenanfangvoriger Beitragnächster BeitragSeitenende Link zu diesem Beitrag

Chris (rothaut)
Suche alle Beiträge dieser Person in dieser Hauptrubrik
Mitglied
Benutzername: rothaut

Nummer des Beitrags: 32
Registriert: 01-2001
Veröffentlicht am Donnerstag, den 02. Januar, 2003 - 03:18:   Beitrag drucken

Hallo,

T kann nicht 7 sein, denn wenn o 3,7 oder 9 ist kann also t nur 1(+5), 3(+5) oder 4(+5) sein. 1,3 und 9 fallen ja weg -> T {4,6,8}
daher ist T also gerade. -> N ist Übertrag von R -> N {2,3,4} -> R>3 also R {4,6,7,8,9}

Jetzt etwas komplizierter :

Ist T=6 folgt, dass O=3 und R=7 oder 9 ist.
Daraus folgt, dass I auch 3 ist. (Wid.)
Also T {4,8} -> I {2,4}

Nu bin ich aber müde, vielleicht kommst Du ja so weiter...

Chris
Seitenanfangvoriger Beitragnächster BeitragSeitenende Link zu diesem Beitrag

Varon (wvvaron)
Suche alle Beiträge dieser Person in dieser Hauptrubrik
Mitglied
Benutzername: wvvaron

Nummer des Beitrags: 18
Registriert: 09-2000
Veröffentlicht am Mittwoch, den 08. Januar, 2003 - 14:38:   Beitrag drucken

Was wäre, wenn T = 0 ist?
Oder ist das nicht im Sinne des Erfinders¿
Auf alle Fälle sind die anderen Variablen dann egal und es ist eine wahre Aussage.

Beitrag verfassen
Das Senden ist in diesem Themengebiet nicht unterstützt. Kontaktieren Sie den Diskussions-Moderator für weitere Informationen.

ad

Administration Administration Abmelden Abmelden   Previous Page Previous Page Next Page Next Page